timmydoeslsat
Thanks Received: 887
Atticus Finch
Atticus Finch
 
Posts: 1136
Joined: June 20th, 2011
 
 
trophy
Most Thanked
trophy
First Responder
 

Seeing inconsistency with PT 30-2-21 and PT 55-3-14

by timmydoeslsat Fri Feb 03, 2012 3:11 pm

I ran across a question today that appeared to be testing the same concept as a previous test. The two items in question are from PT 30-2-21 and PT 55-3-14.

PT 30-2-21

Premise: Leaders of all major parties oppose bill

Conclusion: Bill will almost surely fail to pass.

Strengthen question.

The two most attractive answers here were A and B.

A) Bills that have not been supported by even one leader of a major party MOST ~passed into law

B) Bills that have not been passed into law MOST ~supported by even one member.

(I realize that there is a shift of language in this answer choice from leader to just member, but for the sake of argument, even if this answer choice would have used the word leader, the logic behind this answer choice is still not right)

To have this conclusion state a probability associated with it based on characteristics of this bill, we need to know something about the characteristics of this bill. For all we know, perhaps even bills that are passed are not supported by even one leader of a major party.

So that is why A is correct.

However, in PT 55-3-14. The correct answer essentially is "B" from above. While this answer choice does not give us an "A" like the one above.

This seems inconsistent. Why would an answer choice not be given in terms of likelihood based on the characteristics we know from this building.
User avatar
 
ManhattanPrepLSAT1
Thanks Received: 1909
Atticus Finch
Atticus Finch
 
Posts: 2851
Joined: October 07th, 2009
 
 
 

Re: Seeing inconsistency with PT 30-2-21 and PT 55-3-14

by ManhattanPrepLSAT1 Fri Feb 03, 2012 8:32 pm

Great question Timmy, and looking out for comparisons is so useful!

The LSAT is actually being consistent on this one. The issue is with the comparison. The difference is that in the example from PT55 there are we are already given the likelihood of certain phenomena (the likelihood that buildings from a certain site would contain limestone as the only stone component; and the likelihood of buildings from the same site would be dwellings).

In the example from PT30 we have a single premise and a single conclusion that we simply need to link in conditional logic. Notice the conclusion is not absolute but says "almost surely" fail to pass. So the quantifier in answer choice (A) works well.

(B) reverses the chain of reasoning
(C) negates the chain of reasoning
(D) undermines the argument
(E) tries to contrapose a "most" statement - does not exist!

For another comparison, check out: PT44, S2, Q21 - It is highly likely that

In this one, statements that mimic answer choices (B) and (C) from the example in PT30 are given in the stimulus to hope to confuse you and the correct answer choice (E) presents a statement that is the negated assumption from the argument (which mimics the assumption from PT30.)

The issue in PT55 is overlapping groups.

Most buildings are made only of limestone.
Most buildings are dwellings.

The question arises, "to what extent are these groups overlapping?"

The argument assumes that there is a large overlap between these two groups, which is supported by answer choice (B). If most of the buildings that are not dwellings are not made solely of limestone, then the groups "made solely of limestone" and "dwellings" should have a larger overlap - thus supporting the assumption.

Hope that helps!
 
timmydoeslsat
Thanks Received: 887
Atticus Finch
Atticus Finch
 
Posts: 1136
Joined: June 20th, 2011
 
 
trophy
Most Thanked
trophy
First Responder
 

Re: Seeing inconsistency with PT 30-2-21 and PT 55-3-14

by timmydoeslsat Sat Feb 04, 2012 1:07 am

Thanks for the response Matt. I suppose my hang up is still with the fact that the way PT 33 went from having a premise MOST X statement, I wanted this one to have a premise MOST X statement.

I would have expected this question from PT 55 to give me this for B:
Most non-local building material buildings are not human dwellings


This is the manner in which it was done on PT 33.
 
timmydoeslsat
Thanks Received: 887
Atticus Finch
Atticus Finch
 
Posts: 1136
Joined: June 20th, 2011
 
 
trophy
Most Thanked
trophy
First Responder
 

Re: Seeing inconsistency with PT 30-2-21 and PT 55-3-14

by timmydoeslsat Mon Feb 06, 2012 5:55 pm

While I would have liked to have seen the above answer choice, I suppose it is the case that the two PT answer choices are different because they differ in the amount that they strengthen.

I would say that PT 30's answer choice strengthens the argument much more than PT 55's answer choice. But nonetheless, it strengthens. Am I correct in this thought?
User avatar
 
ManhattanPrepLSAT1
Thanks Received: 1909
Atticus Finch
Atticus Finch
 
Posts: 2851
Joined: October 07th, 2009
 
 
 

Re: Seeing inconsistency with PT 30-2-21 and PT 55-3-14

by ManhattanPrepLSAT1 Thu Feb 16, 2012 4:42 pm

Hey Timmy, I haven't forgotten about this question, it's a good one.

But I'm wondering how do you see these arguments as similar. The example from PT30 has only one premise (which does not involve a "most" statement). Whereas the example from PT55 has at least 3 premises (2 of which involve "most" statements) and seems to be more about overlapping sets.

I guess I don't see why you would expect a similar answer from these two questions...